Límite clásico en la demostración de la mecánica cuántica

Varias preguntas son sobre el límite. 0 , p.ej

Yo leo 0 en mecánica cuántica . Las respuestas altamente votadas dicen que este límite es una forma aceptable de recuperar las leyes de movimiento de Newton de la ecuación de Schroedinger (SE) ( https://physics.stackexchange.com/a/108226/307786 ), y que no lo es ( https ://física.stackexchange.com/a/42007/307786 ). ¿Alguien puede proporcionar una prueba en lugar de ejemplos?

No sé exactamente qué enunciado observable estoy considerando en el límite. Algo fácil, con suerte. El primer enlace que tengo establece que en el límite, el espectro de energía del oscilador armónico cuántico se vuelve continuo. aceptaré una prueba de que

  • todos los estados ligados para cualquier V volverse continua por debajo del límite.

  • O una prueba de que la función de onda toma un significado clásico diferente en el límite.

  • O que SE se convierte en una ecuación de Euler-Lagrange (EL) o una ecuación de Hamilton o una ecuación de Newton ( F = metro a -like) sin números complejos.

  • O que las soluciones de SE parecen funciones delta en el espacio de posición y en el espacio de momento simultáneamente (porque en el centro de masa (CM) no hay incertidumbre de posición ni de momento).

Obtuve estas ideas de ¿Qué hace que una teoría sea "cuántica"? . Creo que una prueba de una de estas afirmaciones implicará la mayoría de las demás. Definitivamente no puedo decir cuál quiero, porque no sé cuáles son correctos y comprobables. Pero aceptaré una prueba para tal argumento.

Considere incluir qué observable / declaración precisa se considera en el límite 0 . La conclusión depende de ello.
@Qmechanic No puedo responder esa pregunta porque no sé qué observable es demostrable (ver edición). Di algunos ejemplos de observables, ¿está bien si obtengo una respuesta para alguno de ellos? gracias.
Metadiscusión relacionada physics.meta.stackexchange.com/q/13713/2451 .
Voté tu respuesta por tu esfuerzo.
El enunciado observable: En el límite clásico es posible medir la posición sin perturbar el impulso, y viceversa. Eso permite encontrar valores agudos de tipo clásico para las distribuciones de posición y momento (pero las funciones de onda siguen siendo de valor complejo).

Respuestas (1)

Voy a responder a esta parte de la pregunta:

O que SE se convierte en una ecuación EL o una ecuación de Hamilton o una ecuación newtoniana (F=ma-like) sin números complejos.

Dejar ψ ser una solución a la ecuación de Schrödinger

i ψ t = 2 2 metro 2 ψ + V ψ

Siempre puedes escribir la función de valores complejos en forma polar ψ = R mi i S / para funciones reales R y S . Si sustituye eso en la ecuación de Schrödinger y separa las partes real e imaginaria, obtiene dos PDE acopladas de valor real

R t = 1 2 metro [ R 2 S + 2 R S ]
y
S t = [ | S | 2 2 metro + V + q ]
dónde
q = 2 2 metro 2 R R
es el "potencial cuántico".

Tenga en cuenta que el único término que tiene un factor de en eso esta q . La primera de las dos ecuaciones, si defines ρ = R 2 = ψ 2 , le dará la ecuación de continuidad de la mecánica cuántica. La segunda es como la ecuación de Hamilton-Jacobi para una partícula clásica con la función principal de Hamilton S excepto por el término "cuántico" adicional q .

Si tomas formalmente 0 , recupera exactamente la ecuación HJ para la partícula clásica, lo que, creo, responde la parte de su pregunta hasta el hecho conocido de que puede ir y venir entre las ecuaciones EL y HJ una vez que está completamente dentro de la mecánica clásica.

(Desde es una constante, en lugar de tomar el límite que va a 0, prefiero pensar en el límite que | q | | | S | 2 / ( 2 metro ) + V | , pero la conclusión es la misma.)

Todavía existe la ecuación de continuidad, que creo que se convierte en parte de su confusión restante. La ecuación de continuidad sigue siendo válida. En el contexto clásico, podría llamarse más a menudo un caso especial de la ecuación de Fokker-Planck, en este caso con difusión 0. Si tiene alguna incertidumbre (en el sentido clásico de no sé todo con precisión en lugar del sentido cuántico de no puedo saber todo con precisión), entonces esta ecuación le dice cómo propagar esa incertidumbre hacia adelante. Lo que es críticamente diferente ahora es que la dinámica subyacente no depende de esa incertidumbre una vez que el q se ignora el término. Si no tiene incertidumbre, lo que está permitido en la teoría clásica, esta ecuación aún funciona en algún sentido generalizado de distribución cuando la distribución llega al límite de una función delta. ρ d .

Algunas de las preguntas, respuestas y documentos a los que hace referencia hacen ese punto explícito y separado del " 0 " límite. Eso podría ser, al menos en parte, una diferencia en la terminología. Estoy dispuesto a llamar a la ecuación de Fokker-Planck "clásica" donde otros quieren darle una etiqueta distintiva como "clásica estocástica" o "clásica probabilística". Los que están en el último campo (correctamente) señalan que también deben tomar el límite. ρ d para llegar al caso clásico "determinista".

¿Por qué es la fase de ψ ¿Sería significativo resolver la ecuación HJ? ¿Cómo relacionas la fase de ψ con algún observable clásico? Si no recuerdo mal la S en la ecuación HJ es la acción clásica con punto final libre
La pregunta, al menos la parte que identifiqué que estaba respondiendo, pregunta cómo reconocer el límite clásico. Eso es todo. La ecuación de Schrödinger tiene una forma real, tal como se da, y bajo las condiciones mencionadas, la forma real deja en claro que la dinámica de la partícula viene dada exactamente por las ecuaciones clásicas. ¿Qué más quieres? @FrodCube Como se señaló en la respuesta, S es la función principal de Hamilton, que tiene relaciones conocidas con un par de variantes de "acción" dentro de la física clásica.
Si sueltas el q término, entonces el lado derecho de la ecuación HJ es la energía total (clásica), que, por supuesto, es un observable clásico. @FrodCube. Una vez que sea clásico, también puede calcular el impulso a partir de eso (ya que también tenemos V ), y con las condiciones iniciales puede calcular la posición en función del tiempo. Esa es toda la física clásica estándar en ese momento, por lo que no entiendo lo que quiso decir con relacionar "con cualquier observable clásico" en el contexto del comentario.
tal vez mi pregunta es tonta. Conozco tu derivación, la he visto en varios libros. Simplemente no tengo una intuición (tal vez lo supe en algún momento) de por qué la fase de una función de onda está relacionada con la acción.
@FrodCube Supongo que prefiero pensarlo al revés. La acción es fundamental, y ponerla en exponencial hace que la definición del operador pag pag ^ = i X ejercicio. La versión compleja de la ecuación es más popular tanto por razones históricas como prácticas, pero (como cuestión de opinión) creo que la versión real está más ligada a los fundamentos intuitivos. Pero de todos modos, probablemente estemos deambulando por una nueva pregunta en este punto...
@Brick Tu respuesta es muy buena.
Si llega a cero, entonces, ¿qué sucede con R ? Es decir, ¿es trivial que el potencial cuántico desaparezca?
@Hulkster, creo que no entiendo tu pregunta. La ecuación para R , como se muestra, no depende directamente de . Hay más discusión sobre eso en el penúltimo párrafo de la respuesta.
De acuerdo. Si R es independiente de ,entonces el límite clásico es claro.